Matemáticas, pregunta formulada por Usuario anónimo, hace 11 meses

Ejercicio 8 a y el b

Adjuntos:

Respuestas a la pregunta

Contestado por josebernad
2

(1/3)³*(1/3)³= (1/3)^6

(-1/2)³*(-1/2)³*(-1/2)³*(-1/2)³= (-1/2)^12

Otras preguntas